404 Not found

GMAT Prep Online Guides and Tips

The 6 gmat essay tips that will help you ace the awa.

gmat awa essays with answers pdf

If you’re preparing for the GMAT, you’ve probably spent countless hours reviewing math concepts and mastering grammar skills. You’ve likely also spent time studying for the newer integrated reasoning section, too. But have you thought about the analytical writing assessment part of the GMAT?

If your answer is no, don’t worry! You’re not alone. Many test-takers go into test day without spending a lot of time preparing for the essay section of the GMAT, especially since it’s unclear how much (or even if) the GMAT essay even matters for getting into business school.

In this article, I’ll shed some light on the oft-forgotten GMAT AWA section. First, I’ll give you an overview of what’s actually on the AWA section. Next, I’ll discuss whether or not that score really matters for your admission to business school. Finally, I’ll tell share  the top GMAT essay tips that are guaranteed to boost your GMAT essay score.

GMAT Analytical Writing Assessment Overview

The GMAT Analytical Writing Assessment is designed to measure your ability to think critically about a topic and then communicate your ideas about that topic. During the AWA section, you’ll be asked to analyze and critique an argument and judged on your ability to do so clearly, thoroughly, and thoughtfully.

The GMAT AWA section consists of one writing task: a 30-minute essay. You’ll complete the AWA portion of the GMAT first, before every other test section.

Not sure how or what to study? Confused by how to improve your score in the shortest time possible? We've created the only Online GMAT Prep Program that identifies your strengths and weaknesses, customizes a study plan, coaches you through lessons and quizzes, and adapts your study plan as you improve.

We believe PrepScholar GMAT is the best GMAT prep program available , especially if you find it hard to organize your study schedule and don't want to spend a ton of money on the other companies' one-size-fits-all study plans.

     Improve Your GMAT Score by 60 Points, Guaranteed     

For your GMAT essay, you’ll be asked to think critically about an argument that’s presented to you. You’re not supposed to give your opinion on the subject itself.

GMAT AWA scores range from 0 to 6, in half-point intervals. Every GMAT AWA response receives two independent scores. According to MBA.com, one of your scores may be performed by an essay-scoring engine. At least one of your GMAT AWA scores will be determined by a GMAT essay reader.

Your AWA score doesn’t affect your GMAT total score and is generally considered the least important of your GMAT scores.

control-427512__340

The 6 Best GMAT Essay Tips

If you’re looking to achieve a GMAT essay score that’ll help you get into business school, these six GMAT Analytical Writing tips will help you achieve success.

#1: Follow the Directions

One of the most important GMAT essay tips is to understand the directions of the AWA section.

The AWA section specifically asks you to critique an argument on its strengths and weaknesses. AWA graders aren’t looking for a well-written, thoughtful opinion piece about the topic discussed in the prompt. They’re looking for you to analyze whether or not the argument itself was sound, and to back up that analysis with evidence from the text, and they’ll judge you on how well you accomplished that specific task. If you don’t follow the directions, you won’t achieve a high score.

#2: Develop a Clear Structure

Another one of the important GMAT writing tips is to take the time to set up your essay in a clear way.

You don’t need to write the most interesting or lengthy essay in the world to score well on the AWA section, but you do need to give your essay an easy-to-follow structure. Usually, that consists of an introduction, three to four well-developed body paragraphs, and a conclusion.

Your introduction should restate the main argument of the prompt, then highlight the flaws in the argument that you’ll discuss in the body of the essay.

Each of the body paragraphs should focus on a specific flaw in the argument. First, you should highlight the flaw itself. Next, you’ll need to explain why that particular flaw is a flaw. Finally, you should highlight how the argument could’ve been made more clearly or more successfully.

In the conclusion, you’ll want to restate each of the reasons why the argument was flawed and summarize how those flaws affected the validity of the argument.

Following this clear, simple structure for your GMAT essay will help you achieve your goal score.

#3: Know the Common AWA Flaws

Your task for the GMAT AWA is to critique an argument given to you in a prompt. That means that you can assume the argument given is a weak one, since your job is basically to analyze its weaknesses.

GMAT AWA prompts typically have arguments that are weak in predictable ways. Be on the lookout for these common “flaws” that you’ll encounter in AWA prompts:

Causality: GMAT AWA prompts often contain errors in causality, which means that they attribute the wrong effect to the wrong cause . If you see an argument that uses causality, make sure you check to make sure that causality is correctly attributed and that there’s a provable causal relationship.

Vagueness: GMAT AWA prompts often contain vague terms or statistics that are used incorrectly to draw conclusions. For instance, a prompt might suggest that, out of a sample of 500 consumers, more are buying name-brand paper towels than generic paper towels. The use of the word “more,” in this case, isn’t specific enough because it doesn’t tell you exactly how many more people are buying name-brand paper towels. You can’t draw a definitive conclusion off of vague data.

Overconfidence: GMAT AWA prompts often contain overconfident language. You should be looking for the language in arguments to be thoughtful and well-balanced. Keep an eye out for words like “undoubtedly,” “definitely,” and “of course,” which indicate overconfidence.

home-office-336378__340

#4: Practice

One of the best GMAT essay tips is to practice, practice, practice before you actually complete the GMAT AWA section on test day. You can find real, retired GMAT AWA prompts on the GMAT website for free. You can also purchase the GMAT Write tool to receive scores on practice AWA prompts if you’re really concerned about your score.

Want to improve your GMAT score by 60 points?

We have the industry's leading GMAT prep program. Built by Harvard, MIT, Stanford, and Wharton alumni and GMAT 99th percentile scorers, the program learns your strengths and weaknesses and customizes a curriculum so you get the most effective prep possible.

Try PrepScholar GMAT for 5 Days Risk-Free.

Practicing will help you in a number of ways. First, practicing will help you master your timing. You’ll only have 30 minutes to craft a logical and well-reasoned essay on test day. The more you practice, the faster you’ll get at outlining and completing your essay.

As I mentioned in the previous GMAT writing tips, you’ll need to fully answer the correct prompt to achieve a good score on your GMAT essay. Practicing will help you get used to the structure of GMAT AWA prompts and help you get used to the types of questions you’ll see on test day.

Finally, practicing will help you get used the structure you need to employ to succeed on your GMAT essay. The more you practice, the more naturally you’ll be able to craft a complete introduction, body, and conclusion for each of your GMAT essays.

#5: Take Time to Outline

While outlining may seem like one of the more basic GMAT essay tips, taking five minutes at the beginning of the AWA section to sketch out a basic outline of your essay will really help you as you start to write.

Everyone outlines differently, but in general, I’d suggest having one to two bullet points for each paragraph that highlight the main ideas the paragraph will cover. Outlining will help you make sure you’ve covered all the main points you need to fully answer the question.

#6: Don’t Sweat the AWA Too Much

The final of my GMAT analytical writing tips is to not worry about the AWA section too much. As I mentioned in a previous section, the AWA section isn’t that important in the overall scheme of your GMAT score. It’d be a mistake to spend a lot of time and energy stressing over and preparing for the AWA section before you take the GMAT.

Spend between three to six hours preparing for the AWA, depending on how comfortable you are writing to the AWA’s structure. More often than not, that’s all the time test-takers need to achieve a solid AWA score.

Your GMAT AWA score won’t make or break your chance of admission to the business school of your dreams. An AWA score between 4-6 will sufficiently demonstrate your writing abilities to most admissions committees, and there’s not a huge advantage to scoring a perfect 6 on the AWA section.

An AWA score of below 4, however, will raise red flags for admissions committees who may question your communication abilities. So, it’s important to study for the AWA section to make sure your score is sufficient.

What’s Next?

Feeling set on GMAT analytical writing tips, but looking for more advice on other sections of the GMAT? We’ve got tons of in-depth, high-quality guides to help you master the content you’ll see on GMAT test day. Check out our guide to the GMAT verbal section to learn how to master the three GMAT question types or read our guide to the GMAT quant section  to understand exactly what math you need to know to achieve your goal GMAT score.

Looking to make an in-depth, comprehensive GMAT study plan? Our guide to GMAT study plans provides four sample study plans that you can adapt to your needs. Pick and choose between one-month, three-month, and six-month study plans that are each designed to boost your GMAT score.

Setting a realistic goal score is a hugely important part of your GMAT prep. By setting a realistic goal score, you give yourself a target to work towards and a benchmark by which to measure your progress as you prep for the GMAT. In our guide to GMAT score requirements, you’ll learn about how to set a goal that makes sense for your abilities and needs as a test-taker.

Was this helpful? Sign up for FREE GMAT and MBA guides!

Share this:.

  • Click to share on Twitter (Opens in new window)
  • Click to share on Facebook (Opens in new window)
  • Click to share on Google+ (Opens in new window)

gmat awa essays with answers pdf

Author: Hayley Milliman

Hayley Milliman is a former teacher turned writer who blogs about education, history, and technology. When she was a teacher, Hayley's students regularly scored in the 99th percentile thanks to her passion for making topics digestible and accessible. In addition to her work for PrepScholar, Hayley is the author of Museum Hack's Guide to History's Fiercest Females. View all posts by Hayley Milliman

gmat awa essays with answers pdf

Homepage > GMAT IR and AWA > GMAT Analytical Writing Assessment tips – How to score a perfect 6 on GMAT AWA

GMAT Analytical Writing Assessment tips – How to score a perfect 6 on GMAT AWA

Posted by Suheb Hussain | Aug 6, 2020 | GMAT Focus Edition , GMAT IR and AWA , GMAT Preparation

GMAT Analytical Writing Assessment tips – How to score a perfect 6 on GMAT AWA

The GMAT Analytical Writing Assessment (AWA) section measures your ability to analyze an argument and communicate your thoughts or ideas. What you simply need to do is to critically analyze the reasoning given behind a given argument. To complete the AWA section of the GMAT you are allotted 30 minutes.

GMAT Analytical writing Assessment

Here is the outline of the article:

How to create a well rounded Analytical Writing Assessment essay?

  • How does ‘GMAT Write’ analyze your AWA essay?

GMAT AWA tips – GMAT Analytical Writing Assessment Template

  • Sample GMAT Analytical Writing Assessment essay

Let’s discuss now how you can score a perfect 6 on the GMAT Analytical Writing Assessment section. Here are a few tips to prepare you for the GMAT AWA section.

Step 1 –  Understand the process of creating a well-rounded analysis of an argument. You can do that in two ways or rather with the help of two tools.

  • AWA template by Chineseburned from gmatclub

Step 2 – Practice! Practice! Practice! the AWA questions. Here is a list of practice questions for you.

GMAT Write – An AWA practice tool by GMAC

GMAT Write is a writing tool provided by GMAC. It gives you access to 2 unique essay prompts and a chance to write 4 essays. The good thing about this tool is that it scores your essay based on the scoring algorithm used by the official GMAT exam. But, it costs $29.99 to subscribe to this tool.

Take a look at this article to know the importance of AWA and IR sections for business school admissions.

How does the ‘GMAT Write’ analyze your GMAT AWA essay?

Once you submit an essay, it scores you on 4 categories:

  • Analysis of the issue
  • Supports ideas
  • Organizes coherent idea
  • Language control

Based on these 4 categories it gives you an overall score for the essay.

Even if you don’t buy this tool, you still get to know what GMAT considers an ideal essay to be from it. An ideal essay should:

  • Identify and analyze significant flaws in the argument
  • Support the critique using relevant supporting reasons and/or examples
  • Be a clearly organized and coherent response
  • Demonstrate control of language, including diction, syntax, and conventions of standard written English

Source: https://www.mba.com/exam-prep/gmat-write

Once you know what factors make a perfect essay, the next thing you should do is creating a template. The most popular template for AWA is the one provided by ‘Chineseburned’ user on the gmatclub forum.

If you are planning to take the GMAT, we can help you with a personalized study plan and give you access to quality online content to prepare. Write to us at  [email protected] . We are the  most reviewed GMAT prep company on gmatclub with more than 1950 reviews. Why don’t you take a free trial and judge for yourself?

Here are the details of his template:

Structure of the essay

  • Introduction: Restate the argument and point out the flaws. Now, state your views which you’ll discuss in the next paragraphs.
  • First paragraph: State your first critique of the argument and support your view with an example.
  • Second paragraph: State your first critique of the argument and support your view with an example.
  • Third paragraph: Pose a few questions for the argument. The absence of information in the argument to answer your questions weaken it further.
  • Fourth argument: State information that you feel would have strengthened the argument but is absent. (This is an extra paragraph which is not in Chineseburned template)
  • Conclusion: State that the argument is flawed because of the above reasons and which reasons could have strengthened the argument.
Learn how the IR section is scored.

Sample GMAT Analytical Writing Assessment AWA essay

Here is a sample AWA essay question for practice. We have also provided the answer to this question based on the template.

The following appeared in an Excelsior Company memorandum. “The Excelsior Company plans to introduce its own brand of coffee. Since coffee is an expensive food item, and since there are already many established brands of coffee, the best way to gain customers for the Excelsior brand is to do what Superior, the leading coffee company, did when it introduced the newest brand in its line of coffees: conduct a temporary sales promotion that offers free samples, price reductions, and discount coupons for the new brand.” Discuss how well reasoned you find this argument. In your discussion, be sure to analyze the line of reasoning and the use of evidence in the argument. For example, you may need to consider what questionable assumptions underlie the thinking and what alternative explanations or counterexamples might weaken the conclusion. You can also discuss what sort of evidence would strengthen or refute the argument, what changes in the argument would make it more logically sound, and what, if anything, would help you better evaluate its conclusion.

GMAT AWA essay

The Excelsior company wants to introduce its own brand of coffee since coffee is an expensive food item and might have good profit margins. However, as there are several established brands of coffee already, it needs to work out a way to gain a foothold in the market. For this purpose, the company plans to pursue the same marketing strategy that the leading coffee company Superior had used. However, I think there are several flaws in that line of reasoning. And hence this argument seems a little weak.

First, Superior is already an established brand and the market leader. No data is given as to how long back it entered the coffee market. Without this information, it is difficult to ascertain the utility of the marketing strategy for Excelsior. A brand might take decades to establish, and what worked a decade back, need not work today. For example, when Google was initially set up, it depended on word of mouth, rather than any communication media, for its marketing. As it was initially just an internal product for the University, this approach worked for them. However, it might not work for any company now. We see these days that most new web companies do advertise heavily when they initially launch a product for the masses.

Second, no information has been provided about the target market for the coffee or the pricing strategy. It might be that the coffee produced by Superior is one of the cheaper products in the market. And that they earn their profits from volume sales rather than having a niche market. Whereas, Excelsior might be aiming at the niche market of expensive and luxury coffee. If that were the case, then the same marketing strategies might not be relevant for Excelsior. Can Hyundai apply the same approach towards the market as does a luxury brand like Ferrari?

Without some of the relevant and contextual details, it is difficult to judge the efficacy of using the same marketing strategy that a competitor applied some unknown years back. If we were to get some more details in this regard, we could have judged the situation better.

Although all the factors being the same, it might not be a bad idea to follow the same approach as that of a leading brand in the market. Sometimes there is nothing wrong with sticking to the tried and the tested. In the absence of any other better idea, Excelsior might not have much to lose if it pursues the same methods as did Superior. It’s better to have some plan rather than no plan.

Conducting a temporary sales promotion that offers free samples, price reductions and discount coupons might not be such a bad idea. It could be a good ploy to get users to try out the product. And discounts and other monetary incentives are usually very effective in this regard. If the quality of the product is good enough and the pricing is appropriate, introducing enough customers to the product through such offers might give a significant push to consumer interest in the product. While customer satisfaction might leverage word of mouth marketing.

Therefore, depending on the context the approach might or might not be a beneficial one. This essay question has been taken from gmatclub and the answer has been judged as a perfect 6 by the GMAT Write tool. The answer was written by the user ‘rish2jain’ from gmatclub forum.

To score a perfect 6 on the GMAT Analytical Writing Assessment section, it is important that you learn the AWA template and practice as much as you can. Just don’t overdo it. It is a good practice to devote 10% of your preparation time to GMAT Analytical Writing Assessment section.

About The Author

gmat awa essays with answers pdf

Suheb Hussain

Don't limit yourself, don't miss these.

gmat awa essays with answers pdf

Helpful posts Curated just for you!

gmat awa essays with answers pdf

You might also like

gmat awa essays with answers pdf

Browse Related Topics

gmat awa essays with answers pdf

Achieve 685+ on the GMAT in 30 days!

Sign up for our free trial and get.

gmat awa essays with answers pdf

400+ Practice questions with detailed solutions

10+ hours of ai-driven video lessons, adaptive mock test with osr+ analysis.

Begin your GMAT Prep today!

The Magoosh logo is the word Magoosh spelled with each letter o replaced with a check mark in a circle.

GMAT Analytical Writing: All About the GMAT Essay and How to Prepare For It

hands on keyboard typing gmat analytical writing assessment

Yup, the rumors are true: you’ll encounter a 30-minute GMAT analytical writing section on test day. But while analytical writing can seem tough at first, finding out exactly what’s expected and how to attack it for a maximum score will do a lot to make the GMAT essay feel manageable! In this post, we’ll take a look at what you need to know to master the GMAT AWA.

Table of Contents

Introduction to gmat analytical writing, what to expect for gmat analytical writing, scoring for gmat analytical writing.

  • How to Approach the GMAT AWA (Strategy and Tips)

Breakdown by Section

Example gmat essays, gmat awa and business school.

You may be thinking: why on earth would the GMAT even want to test my writing skills? In the modern global business world, you will always have contacts whom you know primarily through writing (email, reports, publications, etc.). Similarly, many people important for your advancement will meet you the first time through your writing. You need to be able to make a strong first impression in your writing, through the arguments you present.

On your GMAT writing assignment, the test will present an argument, often in the context of a newspaper editorial or the statement of a company. The nature of this argument will generally allow you to argue for either side, and the side you choose does not affect your score. You will have 30 minutes to read the prompt and construct your essay. Later, your essay will be graded by both a computer and a person on a scale of 0 to 6; your AWA test score will be an average of these two scores.

Don’t forget the main purpose of this task: to measure your GMAT analytical writing skills. This means that whether you argue for or against the argument, your job is to analyze the argument. You’ll need to consider questions such as:

  • What are the assumptions of the argument, and how strong are they?
  • What sort of facts would strengthen or weaken the argument?
  • Are there alternative explanations or perspectives that would explain the facts in question better?

back to top

Both the computer and the human reviewer are looking for particular elements in a good GMAT AWA essay. In short: a successful Analysis of an Argument essay will be clear and cogently argued; it will present the individual critiques in a logically consistent order; it will identify all the points in need of consideration, and it will use word choice and variety of syntax to effectively communicate.

A lot of students wonder: Are spelling, grammar, and punctuation important on the GMAT analytical writing? Will they make or break your score? They are pretty important—luckily, your GMAT SC correction practice will serve you well in this regard. But you can’t leave these elements up to chance: practice correcting your essays (more on this below), and always, always leave a few minutes at the end of the section to proofread your essay.

Remember, you’ll only have 30 minutes for the entire essay. This means that, while you’ll spend the bulk of your time actually writing, you should still set aside around 3-5 minutes at the beginning of the task for planning, then leave another 3-5 minutes at the end to read over and make minor corrections to what you’ve written. These seemingly small actions can have a big impact on your score! But practicing—and following the instructions—is key to mastering the pacing here.

AWA Directions

Throughout the GMAT, knowing the directions ahead of time gives you an edge because you don’t have to spend time reading them on test day. This advantage is compounded on the AWA test section because the instructions are substantial: it’s a lot to read, so it’s that much less to read on test day. Moreover, the “post-argument” paragraph enumerates skills that will be important to practice and master, so you walk into test day armed and ready with your “analysis toolbox” prepared.

With that in mind, here are the directions that precede every AWA argument task:

gmat awa essays with answers pdf

The first bullet point tells us: a good AWA essay is well-organized, has a natural flow from point to point, and is clear and unambiguous about what it is saying. Those are all important points to keep in mind.

Improve your GMAT score with Magoosh.

The second bullet point reminds us: what they present will be, in all likelihood, a flawed argument, but what you must create is a cogent and clear argument, and that will necessarily involve providing clear and relevant support. It’s not enough simply to assert something badly: you must provide justification for what you are saying.

The final bullet points may appear enigmatic: “control the elements of standard written English.” What does that mean?

Well, first of all, it means no grammar or syntax mistakes . It also means varying the sentence structure —some simple sentences (noun + verb), some with two independent clauses (noun + verb + and/but/or + noun + verb), some with dependent clauses, some with infinitive phrases, some with participial phrases, etc. Finally, it means choosing the right words and the right tone : the tone should be skeptical toward the prompt argument and persuasive toward the points you are making, but not arrogant or dogmatic in any way.

The following paragraph always appears after the argument prompt. This is the real meat-and-potatoes of the AWA directions:

gmat awa essays with answers pdf

First of all, notice it give you one clear task: “Be sure to analyze the line of reasoning and the use of evidence in the argument.” Then, it lists several strategies that you might employ in your analysis. Don’t feel compelled to use every one of these in every AWA essay, though you should be using most of them in most essays.

If you’ve only taken standardized tests like the ACT or the SAT before, you may find AWA scoring slightly strange. GMAT Analytical Writing affects your overall score differently than essays on other exams do. With that in mind, let’s take a look at what a good GMAT analytical writing score on the GMAT is, as well as how important AWA test scores are to your overall GMAT score.

The GMAT Analytical Writing Scoring System

First, let’s get an important point out of the way: The AWA test score does not affect your overall GMAT score . Instead, it’s included as a separate category on your score report.

Although you won’t have an exact breakdown of your scores for each element, the GMAT analytical writing assessment is looking at your abilities in roughly four categories:

  • Quality of Ideas
  • Organization
  • Writing Style
  • Grammar and Usage

Based on your overall work, you’ll receive a score from 0 to 6, in half-point increments.

So just who decides where your essay falls within each of these four categories? First of all, a computer does. This is a bit surprising to most students when they first learn about it—after all, how can a computer evaluate something as subjective as writing? Well, remember that you’re not being scored on poetry here. Instead, the computer looks at the organization, syntax, and analytical aspects—things the GMAT algorithm is pretty good at doing.

Then, a trained evaluator will score the essay based on the general development of your ideas and written expression. The GMAT then averages these scores into your overall AWA score. Don’t worry about huge discrepancies in scores between the human and the computer graders, though: if the scores have more than a one-point difference, another human grader comes in to help set the final score.

GMAT AWA Percentiles

If you’re wondering, for example, how a GMAT analytical writing score of 4.5 compares to other test-takers’ scores, you’ll want to look at the GMAT AWA percentiles . GMAC regularly updates this information so that you can see what percentage of previous test-takers received each score.

How to Score Your Practice GMAT Essays

Scoring your own GMAT writing can be a little bit like trying to scratch your own back: it’s hard to see exactly where you are. But it can be done! By breaking down each component of your sample AWA essay, grading it, and averaging those grades, you can get some idea of your strengths and weaknesses. Magoosh even has a GMAT AWA scoring rubric you can use for this purpose.

How to Approach the GMAT AWA

Once you know what to expect from GMAT analytical writing, it’s time to start implementing strategies that will help you maximize your score on this section. Keep coming back to these throughout your GMAT prep to ensure that you’re staying on track and pushing your GMAT writing to the next level!

Strategies for the AWA

Here are the tips that will support your success on the GMAT’s AWA:

  • Recognize Unstated Assumptions : Recognizing assumptions is essential for the Critical Reasoning questions, and it will also serve you well on attacking the prompt argument in your AWA.
  • Know the Directions : This a matter not only of knowing what they say but also, more importantly, understanding the various options you have for analyzing the argument. This list of analytical strategies is always given in the paragraph that follows the prompt argument. It’s important to get familiar with this “analytical toolbox”, so it is yours to employ on test day.
  • Recognize the Common Flaw Patterns : GMAT AWA prompt arguments often contain one of six types of flaws . Learn to spot these patterns, so you are ready on test day.
  • Plan Before You Write : This is obvious to some test-takers. Your first task is to find objections to and flaws in the prompt argument. Create a list of flaws. Then, select the 2-4 of those that are most relevant, that would be the most persuasive talking points. Once you have your list of insightful flaws, then you are ready to write.
  • Paragraph #1: State that the prompt argument is flawed. Briefly enumerate the flaws you will examine, in the order that you will discuss them.
  • Paragraph #2 (or #2 & #3): Sticking to that same order, analyze each flaw in detail, explaining your reasoning why each is a serious weakness of the argument.
  • Last Paragraph: Suggest improvements, which are the reverse of the flaws (i.e. “This argument would be considerably stronger if it did such-and-such to remove flaw #2.”). Close by restating that is it a weak argument.
  • Simple sentence, one independent clause: Jack went to town .
  • Sentence with two independent clauses: Jill went to town and Jack stayed home . (Two independent clauses can be joined by “and”, “or”, “but”, “yet”, “so”, etc.)
  • Sentence with an independent clause and one (or more) dependent clauses: Jack went to the town where Jill lives .
  • Sentence with an infinitive phrase: Jack went to that town to see Jill .
  • Sentence with a participial phrase: Hoping to see Jack, Jill went to town .

A good essay should not have two sentences in a row with the same structure.

  • Vague Language: The words “few”, “many”, “more”, “less”, and “some”, by themselves without numerical qualification, can be vague. Always consider the range of possibilities contained in vague words comparing quantity or size.
  • Inappropriate Comparisons: This form presents a premise and conclusion for Thing #1, which is often quite clear and undisputable. Then, it argues, Thing #2 is very similar, so the premise and conclusion should apply to Thing #2 as well. Depending on the situation, the comparison may not be apt, and pointing out Thing #2 differs from Thing #1 in ways relevant to the argument can expose an essential flaw.
  • Cause/Effect Errors: Many arguments want to make the case that “A causes B.” Whenever the argument “A causes B” is presented, some alternative interpretations to consider are (1) the reverse, “B causes A”; or (2) “A and B are both caused by new thing C”, or (3) “A and B, for a variety of reasons, often appear together, but one does not cause the other.” (This last interpretation is summed up succinctly in the sentence: “Correlation does not imply causality.”) Learn to spot arguments that draw conclusions of causality, and questions whether that’s the correct relationship.
  • Overconfident Conclusions: Confidence = good. Overconfidence = bad. If you read the NY Times or the Wall Street Journal or the Economist magazine, you will notice the kind of tone the GMAT favors: thoughtful, balanced, and measured. Extreme conclusions are seldom correct on the GMAT. Any AWA prompt that presents a conclusion with God-given certainty is too strong, and this is a flaw that needs to be addressed.
  • Proofread! Proofread! Proofread! : When you proofread, you have to consider several levels simultaneous: Is every word spelled correctly? Is every structure grammatically correct? Does the argument logically flow? Unfortunately (or fortunately!) you are not allowed to read your essay aloud in the testing center. What I do recommend, though: silently mouth the words, as if you are carefully pronouncing each word, even though you are not making any sounds. When you move your mouth & tongue, you are engaging more of your brain than when you are simply reading silently with your eyes, and you are more likely to catch subtle mistakes.

Those AWA test tips are all important to keep in mind for your GMAT writing. But when you actually sit down at the computer on test day, what should you do? Here’s the process to use to get the most out of your 30 minutes with the GMAT AWA, including a more in-depth GMAT writing template!

By the time you sit down on test day, you should have read the directions to the AWA (they’re posted above—take another look!), so you won’t need to waste time reading them again. Instead, dive straight into AWA brainstorming . As you brainstorm, list the argument’s flaws; then evaluate those flaws to find which objections are the strongest.

Write an Introduction

You don’t need to reinvent the wheel with each GMAT AWA introduction . Start by stating where the passage is from. Then, focus on two main tasks: summarizing the argument and stating why it’s flawed. Keep it short and sweet; three sentences are enough to get your main points set up!

Construct Your Body Paragraphs

These will make up the lion’s share of your essay, so you’ll spend most of your time writing body paragraphs. Here’s how to go about doing that:  

  • Identification: Focus on a Premise: The first thing you will need to do in your paragraph is to identify what part of the argument you intend to analyze. The best way to do this is by simply summarizing the premise in the argument. You can state that it is flawed at this point, but it is not necessary. You’ll have plenty of time to do that, and the reader already knows what will happen from what you told them in the introduction. Paraphrase, summarize, and use synonyms to present the premise—don’t copy word for word—and this is a great way to lay the groundwork for your analysis.
  • State the Obvious: Flaws Hurt Arguments : This is a common step skipped in student essays. We must return to the larger picture. Students assume that everything will make sense once the flaw is exposed, but this is far too brash. We can’t just expect our reader to “get it.” We need to speak plainly and directly about how the flaw weakens the argument, and more specifically, the recommendation, plan, or conclusion of the argument. Not all flaws weaken arguments in the same way so be specific about what aspect of the conclusion is questionable.
  • Do Good: Improve and Strengthen the Argument : Now that you’ve taken the time to analyze the argument, break down a flaw, and explain the result of that flaw to the conclusion, time to build it back up. Approach the essay as a concerned and interested party, responding to the argument with sympathy. Don’t just be destructive. Give suggestions for improvement. And if you don’t like the conclusion, peer into the heart of what it is trying to accomplish and recommend a way to get there.

Conclude the Essay

First of all, keep in mind that you should not dwell in the conclusion. The heart of your essay, what really matters toward your score, is in the body paragraphs. These should be bulky and in-depth, but the conclusion should be short and to the point. Wrap things up in a timely manner so that you can get to the business of editing and revising your essay.

To keep things manageable and short, don’t go into the details. You only need to recap the major problems in the argument. Sometimes it is enough to say that there are major problems in the argument. Ignore the desire to repeat all the main points that you covered in the body paragraphs. This will only take extra space and waste precious time.

Finally, recommend a way to achieve the goal stated in the article. It is important to approach the analysis of the argument as an interested party. You don’t want to be wholly negative. For one, you will write a better analysis if you imagine yourself tied to the argument in some way, and two, the prompt asks you to strengthen the argument. Find some general evidence that will make the argument more convincing or make it irrefutable. Suggest a change so that the logic stands on firmer ground.

A GMAT analytical writing sample essay, whether well done or flawed in itself, can help you polish your own GMAT writing and bring your essays to the next level. The important part of reviewing example GMAT essays is in analyzing them (and expert analysis is even more helpful, particularly at the beginning).

Where can you find sample GMAT analytical writing prompts? Easy! The GMAC (the GMAT test-maker) actually provides all possible AWA essay topics on their website. So if you need examples of analytical questions for the GMAT , look no further!

So just how important are the AWA scores for business school admissions? We certainly could argue that the GMAT Analytical Writing score is not so important. It’s undeniable that the Quantitative sections and Verbal sections, which contribute to the overall GMAT score, are considerably more important than the separate GMAT writing score . Arguably, the fact that the AWA section was “cut in half” when IR was added in 2012 is a further indication of the relative importance of the GMAT essay and its score.

It’s true that Business school adcoms rely on the Quant, Verbal and Composite scores significantly more than the GMAT writing score. In fact, recent evidence suggest that adcoms also rely on the IR score significantly more than the GMAT essay score.

But while it’s true that, in your GMAT preparation , Quant and Verbal and even IR deserve more attention than the AWA, it’s also true you can’t completely neglect AWA. The difference between a 5 or 6 as your GMAT Analytic Writing score will not make or break a business school admission decision, but having an essay score below a 4 could hurt you.

The purpose of the AWA is to see how well you write, how effectively you express yourself in written form. This is vital in the modern business world, where you may conduct extensive deals with folks you only know via email and online chatting. Some of your important contacts in your business career will know you primarily through your writing, and for some, your writing might be their first experience of you. You never get a second chance to make a first impression, and when this first impression is in written form, the professional importance of producing high-quality writing is clear.

While you don’t need to write like Herman Melville, you need to be competent. A GMAT Analytic Writing score below 4 may cause business schools to question your competence. That’s why it’s important to have at least a decent showing in AWA.

For Non-Native English Speakers

In particular, if English is not your native language, I realize that this makes the AWA essay all the more challenging, but of course, a solid performance on the AWA by a non-native speaker would be a powerful testament to how well that student has learned English . Toward this end, non-native speakers should practice writing the AWA essay and try to get high-quality feedback on their essays.

Devoting 30% or more of your available study time to AWA is likely unwise, but devoting 0% to AWA might also hurt you. Between those, erring on the low side would be appropriate. If, in a three-month span, you write half a dozen practice essays , and get generally positive feedback on them with respect to the GMAT standards, that should be plenty of preparation.

The GMAT analytical writing can feel like a slog when you first encounter it: it requires deep focus and analysis, and it’s not what most students have spent their prep time working on. But with a bit of preparation, your GMAT essays can take your admissions file to the next level by boosting your AWA test score significantly!

By including GMAT writing in your overall GMAT prep schedule, you’ll ensure that this section of the test doesn’t become a drag on your application—and helps, rather than hurts, your shot at your dream school. Good luck!

Rachel Kapelke-Dale

Rachel is one of Magoosh’s Content Creators. She writes and updates content on our High School and GRE Blogs to ensure students are equipped with the best information during their test prep journey. As a test-prep instructor for more than five years in there different countries, Rachel has helped students around the world prepare for various standardized tests, including the SAT, ACT, TOEFL, GRE, and GMAT, and she is one of the authors of our Magoosh ACT Prep Book . Rachel has a Bachelor of Arts in Comparative Literature from Brown University, an MA in Cinematography from the Université de Paris VII, and a Ph.D. in Film Studies from University College London. For over a decade, Rachel has honed her craft as a fiction and memoir writer and public speaker. Her novel, THE BALLERINAS , is forthcoming in December 2021 from St. Martin’s Press , while her memoir, GRADUATES IN WONDERLAND , co-written with Jessica Pan, was published in 2014 by Penguin Random House. Her work has appeared in over a dozen online and print publications, including Vanity Fair Hollywood. When she isn’t strategically stringing words together at Magoosh, you can find Rachel riding horses or with her nose in a book. Join her on Twitter , Instagram , or Facebook !

View all posts

More from Magoosh

GMAT Accommodations: GMAT Extended Time and More

Leave a Reply Cancel reply

Your email address will not be published. Required fields are marked *

gmat awa essays with answers pdf

How to write your GMAT AWA Essay [Effectively]

gmat awa essays with answers pdf

INTRODUCTION

Hello there!

If you’ve found your way to this blog, we assume that you have begun to think about that often-neglected section of the GMAT – the AWA!

If wishes could come true, we’re guessing that GMAT aspirants would have wished away the AWA section away by now!

The AWA involves staring at a blank page and cursor and thinking up stuff to write, not an easy task for many people.

It also consumes a lot of mental bandwidth at the very beginning of the test.

To top it off, it does not even add to your final score!

We can understand why it isn’t your favorite section on the GMAT 🙂

But here’s the good news! There are not many shortcuts to mastering GMAT Quant and Verbal, but there are shortcuts to cracking the AWA section.

By the time you finish reading this guide, you will know what these hacks are. You will learn how to write an effective AWA essay that gets you a good score and leaves you charged for the real test that lies ahead.

This blog will teach you –

  • What you need to know BEFORE you start preparing for the AWA
  • How to use the 30 minutes allotted to AWA to maximum effect
  • How to use a template to make the AWA writing process simpler

Besides this, you will find 8 sample AWA essays to observe and learn from.

Happy reading! 🙂

7 Things to remember before you start prepping for the AWA

gmat awa essays with answers pdf

Before you dive into AWA preparation, there are a few things you should know about the AWA. Many of these facts will ease your AWA fears and bring a smile to your face!

1. Why is the AWA section on the GMAT?

Each section of the GMAT is carefully constructed ( at the expense of millions of dollars, we kid you not!), to test your readiness for an MBA program and for your post-MBA career. One of the skills you will definitely need post-MBA is an ability to analyze an argument impartially and convey your perspective clearly.

This is what the AWA tests you on.

2. On the AWA, you need to be a lawyer, that is, you need to find faults with the given argument. You do not need to be a journalist, that is, you do not need to write about all possible perspectives of an issue.

Also remember, that this is an analysis, not an opinion piece. Do not bring YOUR perspective and your opinions into the essay. Your only goal is to analyse the given argument.

3. Unlike the Quant and Verbal sections, where your thumb-rule should be to get as high a score as possible, we’d suggest that you do not expend too much mental energy on the AWA, trying to score a 6 on 6.

Getting a perfect 6 will look good on your GMAT score card, and will sound great as you’re telling your friends about it. But it will not be the make-or-break factor in your application. A 4 or a 5 is good enough.

4. The AWA is graded by an E-reader application and by a human reader. Since there is an element of automated grading, you can rig the test to an extent. We’ve found that if you write a substantial essay of over 500 words, and if you structure the argument well ( check our CrackVerbal template in the following chapter), you are almost guaranteed to get a 4+ score!

5. Assuming that you prepare for the GMAT over a course of three months, we recommend that you practice writing 5 to 10 essays, and make sure you get feedback for all of them. If you cover this much practice ground, you’re good to go!

6. One of the best things about the AWA section is that you know all of the questions beforehand (yes, they’re all up there on the GMAC site – Analytical Writing Section ). So you do not have to go hunting for ‘authentic’ AWA essay questions.

7. Now you can choose the order in which you want to take up the sections before starting the test. It is advisable to keep in mind the order that would be helpful for you and prepare for the AWA based on that strategy. This is a recent change to the GMAT test structure. It was introduced in July 2017. We have done a detailed analysis of what this means to an Indian GMAT test-taker in the this blog

8. We saved the coolest point for last 🙂

The AWA lends itself very easily to the use of an essay template. No matter what the argument prompt is, you can bet that there will be at least 3 glaring errors of logic in it. You can, therefore, use a template to structure your AWA essay. Using a template takes most of the stress away from the AWA section.

In the few minutes before you start, you can jot down the template on your scratchpad, so that you don’t have to remember it anymore. Also, because you can plan many of your sentences beforehand, you can get at least a 100 words down before you even read the question!

There are a lot of templates on the internet – probably the most famous one being the Chineseburned AWA template.

At CrackVerbal, we have our own template for the AWA, a modified version of the Chineseburned template. We call it the CrackVerbal AWA Template on Steroids! 🙂

The AWA Writing Process

MBA Essay Writing

1. Write your templatized response

This should take you about 5 minutes:

Type out your prepared template response. Below is a sample. We definitely do not recommend that you use the same words. What you can do, however, is read a few templates on the net, and then write your own. Since you have written it yourself, it will be that much easier to memorise it.

CRACKVERBAL AWA TEMPLATE

The argument claims that < restate the argument >. Stated in this way the argument fails to take into account a few key factors which could call the conclusion to question. It rests on some assumptions, for which there is no clear evidence. Therefore, the argument is unconvincing and falls apart at the seams.

Paragraph 1:

1. Firstly, ( ) 2. This statement is a stretch and not substantiated in any way. 3. The argument would have been much clearer if ( )

Paragraph 2:

1. Second ( ). This is again a very weak and unsupported claim as the argument ( ). 2. For example, 3. This argument would have sounded a lot more convincing if 4. In addition, it would have been strengthened ever further if the argument provided evidence that

Paragraph 3:

1. Finally, the argument concludes that 2. However, what is not clear here is ( ) 3. If there had been evidence to support ( )

In summary, the argument fails to convince because of the faulty assumptions aforementioned. If the argument had drawn upon examples as suggested, and thereby plugged in the holes in the reasoning, it would have been far sounder on the whole.

2. Brainstorm

Now that you have put the pre-planned portion of the essay down, it’s time to read the AWA prompt and wear your thinking hat. GMAT, in its politically correct, non-partisan way, says ‘Discuss how well-reasoned you find this argument’. Remember however, that an AWA argument is never well-reasoned!

There are always a couple of glaring flaws in logic you can pounce on. If these flaws do not occur to you immediately, because of test-day stress, do not assume that you have been given a particularly sound argument. There is no such thing on the AWA!

If you’re unable to be critical, imagine that the author of the argument is somebody you dislike..a teacher you hated at college, or that guy who overtook you and almost dented your car this morning! There, now you’re in the right frame of mind to attack the argument 🙂

Before you do so, you need to understand the three elements of the argument – Conclusion, Premise and Assumptions.

Let us look at an example, and detect these three elements.

“Most companies would agree that as the risk of physical injury occurring on the job increases, the wages paid to employees should also increase. Hence it makes financial sense for employers to make the workplace safer: they could thus reduce their payroll expenses and save money.”

The conclusion is the decision/statement that the author has arrived at. In this case, the conclusion is the last sentence – “Hence it makes financial sense for employers to make the workplace safer: they could thus reduce their payroll expenses and save money.”

The premises are the building blocks of facts on which the conclusion rests. In other words, a premise is what is offered as support for the conclusion. In this case, the premise is – Most companies would agree that as the risk of physical injury occurring on the job increases, the wages paid to employees should also increase.

Assumptions are the unstated, unwritten premises that plug the gap between the written premises and the conclusion. It is the assumptions that you need to attack on the AWA!

How do you attack assumptions?

Remember that an assumption can be incorrect for a variety of reasons. Here are a few types of incorrect assumptions –

1. The Sampling Assumption – The sampling argument assumes that a small group is representative of a much larger group to which it belongs.

2. The illogical analogy assumption – The illogical analogy states that because something applies to A, it applies to B also.

3. The Causal Assumption – The Causal Assumption confuses correlation with causation. That means, just because ‘A’ usually occurs after ‘B’ occurs, does not necessarily imply that B happens because of A.

4. The Data Bias – This occurs when the data for a statistical inference itself is drawn from a sample that is not representative of the population under consideration. This is a case of faulty data leading to faulty assumptions.

5. The Non Sequitur – This simply means, finding a connection where there is none. Non Sequitur means “does not follow,” which is short for: the conclusion does not follow from the premise.

Don’t let these categories overwhelm you. We’ve put them down here to get you thinking. However, you can find faulty assumptions with ease, even if you have no clue what a non sequitur is!

As you brainstorm, you will need to jot down your thoughts on the scratchpad. Keep it crisp and brief. Make sure you have these things down –

1. Conclusion + Premise: 2. Flawed Assumption #1: 3. Flawed Assumption #2: 4. Flawed Assumption #3:

For each assumption, also make a cursory note of why it is flawed, an example that talks about why it is flawed, and what additional data would strengthen the argument ( or if you are convinced that you can remember these additional details without having to make a note of them, you can get on with the writing! )

This should take you about 15 minutes:

Here is where you fill in your templatized response with specific details.

The only detail you need to add to the first paragraph is a summary of the argument that is presented. In the above template, your summary should go here ->

1. Start off by pointing out the first flawed assumption. 2. Explain why this assumption is flawed. 3. Give an example that supports the flaw. 4. Explain what further information could have strengthened this argument.

1. Start off by pointing out the second flawed assumption. 2. Explain why this assumption is flawed. 3. Give an example that supports the flaw. 4. Explain what further information could have strengthened this argument.

Paragraph 4:

1. Start off by pointing out the third flawed assumption. 2. Explain why this assumption is flawed. 3. Give an example that supports the flaw. 4. Explain what further information could have strengthened this argument.

Paragraph 5:

This is the concluding paragraph. You already have it down in your template! 🙂

4. Proofread

Are you wondering if three minutes is really enough time to proof-read a 500 word essay?

Here’s the deal – The AWA section is about whether you can analyse an argument and discuss it in an articulate manner. It is not a test of grammar and spelling. Hence, the GMAT will excuse minor errors in spelling and grammar.

However, you should understand that a human reader is going to be reviewing your work, and any human reader will have an unconscious bias against bad grammar and spellings. Hence, you want to keep your essay as error-free as possible, without worrying about it too much.

Three minutes should be able time for you to quickly glance through the document and make sure you haven’t made any obvious errors.

Voila! 🙂 Your AWA essay is ready!

Also Read: GMAT Section Selection – Everything you need to know

Sample AWA Essays

gmat awa essays with answers pdf

Sample Essay 1

”Most companies would agree that as the risk of physical injury occurring on the job increases, the wages paid to employees should also increase. Hence it makes financial sense for employers to make the workplace safer: they could thus reduce their payroll expenses and save money.”

Discuss how well reasoned you find this argument. In your discussion be sure to analyze the line of reasoning and the use of evidence in the argument.

For example, you may need to consider what questionable assumptions underlying the thinking and what alternative explanations or counter examples might weaken the conclusion. You can also discuss what sort of evidence would strengthen or refute the argument, what changes in the argument would make it more logically sound, and what, if anything, would help you better evaluate its conclusion.

Introduction:

This argument states that it makes financial sense for employers to make the workplace safer because lower wages could then be paid to employees. This conclusion is based on the premise that as the risk of physical injury increases, the wages paid to employees should also increase. However, this argument makes several unsupported assumptions. For example, the argument assumes that the costs associated with making the workplace safe do not outweigh the increased payroll expenses due to hazardous conditions.

Body Paragraph 1

The first issue to be addressed is whether increased labor costs justify large capital expenditures to improve the work environment. Clearly one could argue that if making the workplace safe would cost an exorbitant amount of money in comparison to leaving the workplace as is and paying slightly increased wages than it would not make sense to improve the work environment. For example, if making the workplace safe would cost $100 million versus additional payroll expenses of only $5,000 per year, it would make financial sense to simply pay the increased wages. No business or business owner would pay all that extra money just to save a couple dollars and improve employee health and relations. To consider this, a cost benefit analysis must be made. I also feel that although a cost benefit analysis should be the determining factor with regard to these decisions making financial sense, it may not be the determining factor with regard to making social, moral and ethical sense.

Body Paragraph 2

Finally one must understand that not all work environments can be made safer. For example, in the case of coal mining, a company only has limited ways of making the work environment safe. While companies may be able to ensure some safety precautions, they may not be able to provide all the safety measures necessary. In other words, a mining company has limited ability to control the air quality within a coal mine and therefore it cannot control the risk of employees getting black. In other words, regardless of the intent of the company, some jobs are simply dangerous in nature.

In conclusion, while at first it may seem to make financial sense to improve the safety of the work environment sometimes it truly does not make financial sense. Furthermore, financial sense may not be the only issue a company faces. Other types of analyses must be made such as the social ramifications of an unsafe work environment and the overall ability of a company to improve that environment (i.e., coal mine). Before any decision is made, all this things must be considered, not simply the reduction of payroll expenses.

Sample Essay 2

The following appeared in a memorandum issued by a large city’s council on the arts.

“In a recent citywide poll, fifteen percent more residents said that they watch television programs about the visual arts than was the case in a poll conducted five years ago. During these past five years, the number of people visiting our city’s art museums has increased by a similar percentage. Since the corporate funding that supports public television, where most of the visual arts programs appear, is now being threatened with severe cuts, we can expect that attendance at our city’s art museums will also start to decrease. Thus some of the city’s funds for supporting the arts should be reallocated to public television.”

Discuss how well reasoned… etc.

Introduction

In this argument the author concludes that the city should allocate some of its arts funding to public television. The conclusion is based on two facts: (1) attendance at the city’s art museum has increased proportionally with the increases in visual-arts program viewing on public television, and (2) public television is being threatened by severe cuts in corporate funding. While this argument is somewhat convincing, a few concerns need to be addressed.

To begin with, the argument depends on the assumption that increased exposure to the visual arts on television, mainly public television, has caused a similar increase in local art-museum attendance. However, just because increased art-museum attendance can be statistically correlated with similar increases in television viewing of visual-arts programs, this does not necessarily mean that the increased television viewing of arts is the cause of the rise in museum attendance.

Moreover, perhaps there are other factors relevant to increased interest in the local art museum; for instance, maybe a new director had procured more interesting, exciting acquisitions and exhibits during the period when museum attendance increased, in addition, the author could be overlooking a common cause of both increases. It is possible that some larger social or cultural phenomenon is responsible for greater public interest in both television arts programming and municipal art museums.

Body Paragraph 3

To be fair, however, we must recognize that the author’s assumption is a special case of a more general one that television viewing affects people’s attitudes and behavior. Common sense and observation tell me that this is indeed the case. After all, advertisers spend billions of dollars on television ad time because they trust this assumption as well.

In conclusion, I am somewhat persuaded by this author’s line of reasoning. The argument would be strengthened if the author were to consider and rule out other significant factors that might have caused the increase in visits to the local art museum.

Sample Essay 3

The following appeared in a report presented for discussion at a meeting of the directors of a company that manufactures parts for heavy machinery.

“The falling revenues that the company is experiencing coincide with delays in manufacturing. These delays, in turn, are due in large part to poor planning in purchasing metals. Consider further that the manager of the department that handles purchasing of raw materials has an excellent background in general business, psychology, and sociology, but knows little about the properties of metals. The company should, therefore, move the purchasing manager to the sales department and bring in a scientist from the research division to be manager of the purchasing department.”

In response to a coincidence between falling revenues and delays in manufacturing, the report recommends replacing the manager of the purchasing department. The grounds for this action are twofold. First, the delays are traced to poor planning in purchasing metals. Second, the purchasing manager’s lack of knowledge of the properties of metals is thought to be the cause of the poor planning. It is further recommended that the position of the purchasing manager be filled by a scientist from the research division and that the current purchasing manager be reassigned to the sales department. In support of this latter recommendation, the report states that the current purchasing manager’s background in general business, psychology, and sociology equip him for this new assignment. The recommendations advanced in the report are questionable for two reasons.

To begin with, the report fails to establish a causal connection between the falling revenues of the company and the delays in manufacturing. The mere fact that falling revenues coincide with delays in manufacturing is insufficient to conclude that the delays caused the decline in revenue. Without compelling evidence to support the causal connection between these two events, the report’s recommendations are not worthy of consideration.

Second, a central assumption of the report is that knowledge of the properties of metals is necessary for planning in purchasing metals. No evidence is stated in the report to support this crucial assumption. Moreover, it is not obvious that such knowledge would be required to perform this task. Since planning is essentially a logistical function, it is doubtful that in-depth knowledge of the properties of metals would be helpful in accomplishing this task.

In conclusion, this is a weak argument. To strengthen the recommendation that the manager of the purchasing department be replaced, the author would have to demonstrate that the falling revenues were a result of the delays in manufacturing. Additionally, the author would have to show that knowledge of the properties of metals is a prerequisite for planning in purchasing metals.

Sample Essay 4

The following appeared in an announcement issued by the publisher of The Mercury, a weekly newspaper.

“Since a competing lower-priced newspaper, The Bugle, was started five years ago, The Mercury’s circulation has declined by 10,000 readers. The best way to get more people to read The Mercury is to reduce its price below that of The Bugle, at least until circulation increases to former levels. The increased circulation of The Mercury will attract more businesses to buy advertising space in the paper.”

A newspaper publisher is recommending that the price of its paper, The Mercury, be reduced below the price of a competing newspaper, The Bugle. This recommendation responds to a severe decline in circulation of The Mercury during the 5-year period following the introduction of The Bugle. The publisher’s line of reasoning is that lowering the price of The Mercury will increase its readership, thereby increasing profits because a wider readership attracts more advertisers. This line of reasoning is problematic in two critical respects.

While it is clear that increased circulation would make the paper more attractive to potential advertisers, it is not obvious that lowering the subscription price is the most effective way to gain new readers. The publisher assumes that price is the only factor that caused the decline in readership. But no evidence is given to support this claim. Moreover, given that The Mercury was the established local paper, it is unlikely that such a mass exodus of its readers would be explained by subscription price alone.

There are many other factors that might account for a decline in The Mercury’s popularity. For instance, readers might be displeased with the extent and accuracy of its news reporting, or the balance of local to other news coverage. Moreover, it is possible The Mercury has recently changed editors, giving the paper a locally unpopular political perspective. Or perhaps readers are unhappy with the paper’s format, the timeliness of its feature articles, its comics or advice columns, the extent and accuracy of its local event calendar, or its rate of errors.

In conclusion, this argument is weak because it depends on an oversimplified assumption about the causal connection between the price of the paper and its popularity. To strengthen the argument, the author must identify and explore relevant factors beyond cost before concluding that lowering subscription prices will increase circulation and, thereby, increase advertising revenues.

Sample Essay 5

The following appeared as part of an article in a magazine devoted to regional life.

“Corporations should look to the city of Helios when seeking new business opportunities or a new location. Even in the recent recession, Helios’s unemployment rate was lower than the regional average. It is the industrial center of the region, and historically it has provided more than its share of the region’s manufacturing jobs. In addition, Helios is attempting to expand its economic base by attracting companies that focus on research and development of innovative technologies.”

In this argument corporations are urged to consider the city of Helios when seeking a new location or new business opportunities. To support this recommendation, the author points out that Helios is the industrial center of the region, providing most of the region’s manufacturing jobs and enjoying a lower-than-average unemployment rate. Moreover, it is argued, efforts are currently underway to expand the economic base of the city by attracting companies that focus on research and development of innovative technologies. This argument is problematic for two reasons.

To begin with, it is questionable whether the available labor pool in Helios could support all types of corporations. Given that Helios has attracted mainly industrial and manufacturing companies in the past, it is unlikely that the local pool of prospective employees would be suitable for corporations of other types. For example, the needs of research and development companies would not be met by a labor force trained in manufacturing skills. For this reason, it’s unlikely that Helios will be successful in its attempt to attract companies that focus or research and development of innovative technologies.

Another problem with the available work force is its size. Due to the lower than average unemployment rate in Helios, corporations that require large numbers of workers would not find Helios attractive. The fact that few persons are out of work suggests that new corporations will have to either attract new workers to Helios or pay the existing workers higher wages in order to lure them away from their current jobs. Neither of these alternatives seems enticing to companies seeking to relocate.

In conclusion, the author has not succeeded in providing compelling reasons for selecting Helios as the site for a company wishing to relocate. In fact, the reasons offered function better as reasons for not relocating to Helios. Nor has the author provided compelling reasons for companies seeking new business opportunities to choose Helios.

Sample Essay 6

The following appeared in the health section of a magazine on trends and lifestyles.

“People who use the artificial sweetener aspartame are better off consuming sugar, since aspartame can actually contribute to weight gain rather than weight loss. For example, high levels of aspartame have been shown to trigger a craving for food bydepleting the brain of a chemical that registers satiety, or the sense of being full. Furthermore, studies suggest that sugars, if consumed after at least 45 minutes of continuous exercise, actually enhance the body’s ability to burn fat. Consequently, those who drink aspartame-sweetened juices after exercise will also lose this calorie-burning benefit. Thus it appears that people consuming aspartame rather than sugar are unlikely to achieve their dietary goals.”

In this argument the author concludes that people trying to lose weight are better off consuming sugar than the artificial sweetener aspartame. To support this conclusion the author argues that aspartame can cause weight gain by triggering food cravings, whereas sugar actually enhances the body’s ability to burn fat. Neither of these reasons provides sufficient support for the conclusion.

The first reason that aspartame encourages food cravings is supported by research findings that high levels of aspartame deplete the brain chemical responsible for registering a sense of being satedHidden text (sated, sating ), or full. But the author’s generalization based on this research is unreliable. The research was based on a sample in which large amounts of aspartame were administered; however, the author applies the research findings to a target population that includes all aspartame users, many of whom would probably not consume high levels of the artificial sweetener.

The second reason that sugar enhances the body’s ability to burn fat is based on the studies in which experimental groups, whose members consumed sugar after at least 45 minutes of continuous exercise, showed increased rates of fat burning. The author’s general claim, however, applies to all dieters who use sugar instead of aspartame, not just to those who use sugar after long periods of exercise. Once again, the author’s generalization is unreliable because it is based on a sample that clearly does not represent all dieters.

To conclude, each of the studies cited by the author bases its findings on evidence that does not represent dieters in general; for this reason, neither premise of this argument is a reliable generalization. Consequently, I am not convinced that dieters are better off consuming sugar instead of aspartame.

Sample Essay 7

The following appeared in the editorial section of a corporate newsletter.

“The common notion that workers are generally apathetic about management issues is false, or at least outdated: a recently published survey indicates that 79 percent of the nearly 1,200 workers who responded to survey questionnaires expressed a high level of interest in the topics of corporate restructuring and redesign of benefits programs.”

Based upon a survey among workers that indicates a high level of interest in the topics of corporate restructuring and redesign of benefits programs, the author concludes that workers are not apathetic about management issues. Specifically, it is argued that since 79 percent of the 1200 workers who responded to survey expressed interest in these topics, the notion that workers are apathetic about management issues is incorrect. The reasoning in this argument is problematic in several respects.

First, the statistics cited in the editorial may be misleading because the total number of workers employed by the corporation is not specified. For example, if the corporation employs 2000 workers, the fact that 79 percent of the nearly 1200 respondents showed interest in these topics provides strong support for the conclusion. On the other hand, if the corporation employs 200,000 workers, the conclusion is much weaker.

Another problem with the argument is that the respondents’ views are not necessarily representative of the views of the work force in general. For example, because the survey has to do with apathy, it makes sense that only less apathetic workers would respond to it, thereby distorting the overall picture of apathy among the work force. Without knowing how the survey was conducted, it is impossible to assess whether or not this is the case.

A third problem with the argument is that it makes a hasty generalization about the types of issues workers are interested in. It accords with common sense that workers would be interested in corporate restructuring and redesign of benefits programs, since these issues affect workers very directly. However, it is unfair to assume that workers would be similarly interested in other management issues—ones that do not affect them or affect them less directly.

In conclusion, this argument is not convincing as it stands. To strengthen it, the author would have to show that the respondents account for a significant and representative portion of all workers. Additionally, the author must provide evidence of workers’ interest other management topics—not just those that affect workers directly.

Sample Essay 8

The following appeared in the opinion column of a financial magazine.

“On average, middle-aged consumers devote 39 percent of their retail expenditure to department store products and services, while for younger consumers the average is only 25 percent. Since the number of middle-aged people will increase dramatically within the next decade, department stores can expect retail sales to increase significantly during that period. Furthermore, to take advantage of the trend, these stores should begin to replace some of those products intended to attract the younger consumer with products intended to attract the middle-aged consumer.”

Based on an expected increase in the number of middle-aged people during the next decade, the author predicts that retail sales at department stores will increase significantly over the next ten years. To bolster this prediction, the author cites statistics showing that middle-aged people devote a much higher percentage of their retail expenditure to department-store services and products than younger consumers do. Since the number of middle-aged consumers is on the rise and since they spend more than younger people on department-store goods and services, the author further recommends that department stores begin to adjust their inventories to capitalize on this trend. Specifically, it is recommended that department stores increase their inventory of products aimed at middle- aged consumers and decrease their inventory of products aimed at younger consumers. This argument is problematic for two reasons.

First, an increase in the number of middle-aged people does not necessarily portend an overall increase in department-store sales. It does so only on the assumption that other population groups will remain relatively constant. For example, if the expected increase in the number of middle-aged people is offset by an equally significant decrease in the number of younger people, there will be little or no net gain in sales.

Second, in recommending that department stores replace products intended to attract younger consumers with products more suitable to middle-aged consumers, the author assumes that the number of younger consumers will not also increase. Since a sizable increase in the population of younger consumers could conceivably offset the difference in the retail expenditure patterns of younger and middle- aged consumers, it would be unwise to make the recommended inventory adjustment lacking evidence to support this assumption.

This argument is unacceptable. To strengthen the argument the author would have to provide evidence that the population of younger consumers will remain relatively constant over the next decade.

We hope that our strategies help you conquer GMAT AWA with enough and more energy to spare for the sections that follow!

Now that you’ve figured out how to tackle the AWA section, do you want to put theory to practice and get your AWA essay graded?

Our experts here at CrackVerbal will evaluate and grade your AWA essay and give you specific, actionable feedback.

gmat awa essays with answers pdf

The GMAT AWA: Understanding the Format and How to Prepare

The GMAT AWA - Understanding the Format and How to Prepare

The GMAT AWA consists of a single long-form essay, and is an assessment of your ability to analyze and critique a flawed argument. It can be quite an intimidating section for many GMAT candidates, particularly those who are non-native speakers. After several sections of relatively straightforward multiple-choice questions , being asked to write a free-form essay can be extremely daunting.

If you understand the format of the GMAT AWA, however, you stand a much better chance of acing it – native speaker or not. In fact, there have been cases of non-native candidates getting 6.0 on the AWA multiple times, simply because they prepped properly and because they knew the GMAT AWA format inside out.

During the course of this article, we’ll talk you through the GMAT AWA. What is it? How is it structured? And how can you prepare for it?

Table of Contents

What is the AWA?

AWA stands for “Analytical Writing Assessment”. The candidate is given a flawed argument, and 30 minutes in which to craft a long-form response to that argument.

The AWA is notable in several ways. Firstly, it’s not multiple choice like all other sections of the GMAT . Because it’s long-form, it’s scored in a different way to the other sections (which are easier to grade; in Quant and Verbal, you’re either right or you’re wrong). It’s actually double-marked – once by a human examiner, and once by computer. If there’s a disparity between those two scores greater than 1.0, a second human assessor is asked to re-mark it.

Secondly, your AWA score is not factored into your overall GMAT score , but is instead a discrete standalone score. It’s scored from 0-6, and is scored in half-point increments (e.g. 0.5, 1, 1.5, etc.). 

Are AWA Scores Important?

AWA scores are often considered to be of secondary importance in comparison to the main GMAT score . And while this is often the case, the AWA should not be completely disregarded. Many business schools do indeed have cutoffs for the AWA as well as the main score – often around 4.0-4.5. It’s therefore a good idea to make sure that you adequately prepare for the AWA, and that you aim for 4.5 or higher to be safe.

How is the AWA Scored?

The AWA rubric takes into account three general areas:

Linguistic Ability

The criterion that is perhaps the most unnerving for non-native speakers, this measures your knowledge of technical written English. It includes such minutiae as spelling, grammar and punctuation.

Examiners are cognizant of the fact that not everybody is a native speaker, and so there is a little leeway. However, it generally pays to be aware of these factors, and to try to ensure that your writing is as accurate as possible.

Presentation and Organization

This criterion assesses your ability to present your arguments in a clear and coherent manner, effectively use transitions, and your introduction, main body and conclusion can be readily identified. The clearer and better laid-out your essay, the higher your score will be here.

Logical Analysis

The assessor will examine the quality of your arguments when considering this portion of the rubric. They will also assess how relevant your supporting arguments and examples are to the wider point you’re trying to make.

What Kind of Arguments Appear on the GMAT AWA?

As previously mentioned, you’re tasked with analyzing and critiquing a “flawed argument” on the AWA. But not all arguments are flawed in the same way, and a big part of critiquing an argument is understanding how, exactly, it’s flawed.

Let’s take a look at some of the ways in which those flaws present themselves.

Unclear Language

This refers to the use of ambiguous or vague wording such as “many”, “more”, “some,” “less” etc. to make a point without reference to substantiating evidence. For example, the assertion “many people prefer hamburgers to hot dogs” is ultimately meaningless without further information. What percentage of people? What is the source of this assertion? What proof is there that this is true?

Sloppy Sample Usage 

This refers to the use of statistics to reach faulty conclusions. It can include such issues as using overly small sample sizes to reach broad conclusions, or using statistics in an inappropriate or misleading manner.

An example of this would be claiming that 70% of a movie’s Google reviews are negative, and therefore the movie was poorly received overall. This does not take into account viewers who didn’t write reviews, and other contextualizing factors e.g. whether or not there was a group of people incentivized to negatively review the movie who skewed the resultant reviews on Google.

Inaccurate or Inappropriate Comparisons

This fallacy involves the comparison of two things with the assumption that they are sufficiently similar for the comparison to be accurate, when this is not necessarily the case. An example would be claiming the following: “cars kill more people than guns, but nobody suggests banning cars!”

The comparison fails to take into account the myriad differences between a gun and a car, both practical (a car’s primary purpose is not to cause injury) and legal (people must take a test to be licensed to operate a car).

Correlation and Causation

 This is a very common logical fallacy, and involves making the assumption that A causes B, rather than accounting for the fact that although A may be related to B, that doesn’t mean it causes it.

An example would be claiming that because people who eat ice cream often get sunburned, then ice cream causes sunburn. While there is a positive correlation between people who eat ice cream and people who get sunburned, this argument misreads the relationship between the two and doesn’t account for the true reason that the two are related – both of these things happen more frequently because it’s sunny.

What are you Required to do in AWA Essays?

You now have an idea of the kind of faulty reasoning typically presented in AWA arguments, but how do you address that faulty reasoning? What is actually required of you when crafting your response to said arguments?

After presenting the argument, an AWA essay will typically ask you to do some or all of the following:

  • Discuss to what extent you find the argument well-reasoned;
  • Assess the argument’s line of reasoning, as well as how well the argument is supported with evidence;
  • Identify and critique faulty assumptions;
  • Offer counterexamples or alternate explanations that weaken, undermine or rebut the original argument;
  • Explore how the original argument might have been made stronger, either by offering logical support or providing evidence that helps better contextualize it.

Preparing for the AWA

The AWA can, as mentioned, seem extremely daunting. There’s no simple “right” or “wrong” answer, and because it’s completely up to you to craft your response, it seems that there’s a lot that can go wrong.

The truth is that the AWA may seem extremely free-form, but your answer is actually tightly controlled by the argument presented, the parameters set after the argument, and the typical structure of an essay. These restrictions will help you to establish a strategy – and inform the nature of your preparation.

Here’s how to best go about your GMAT AWA preparation.

10 Tips to Prepare for the GMAT AWA and Nail it

Ensure you Fully Understand the Directions  

It’s impossible to start working on your essay if you don’t fully comprehend what is expected of you. That’s why it’s imperative that you carefully read over the paragraph that follows the stated argument, and ensure that you know what you need to do in order to construct your response essay in the manner required.

It’s a good idea to identify any of the phrases mentioned in the “what are you required to do…” section above, as these will assist you in figuring out how to answer the question.

What does this mean when you’re preparing for the AWA? It means that you should check out plenty of practice questions and use the directions when crafting your response. You can then compare your response to top answers submitted by other, successful candidates, or submit your response for peer evaluation on GMAT forums.

Work On Recognizing Assumptions

A key skill in answering AWA questions well is figuring out the assumptions inherent to the faulty arguments you’re being asked to critique. The assumption is the gap between the premise of an argument (the starting point) and the conclusion.

Consider the argument: “Company A is spending twice as much on advertising as competitors, and will therefore see an increase in customers in coming months”. The assumption here is that advertising expenditure directly leads to an increase in customers, which is not necessarily true and can be weakened by your critiques or evidence to the contrary.

If you can nail identifying the assumption, you’re halfway towards deconstructing it – and acing your AWA essay .

Memorize the Different Types of Flaws

The flaws we covered earlier are the most common types that you’ll see cropping up on the AWA. It’s therefore a good idea to thoroughly familiarize yourself with all of them; that way, you’ll be able to quickly identify them on test day, and get to writing your rebuttal more quickly.

Establish a Basic Essay Template and Practice Using it

A handy tip for your GMAT AWA preparation is to drill essay-writing into your head (and hand) so thoroughly that it becomes second nature for you to produce them. If you have a solid scaffold for essay-writing firmly entrenched in your head, you don’t need to waste time in your test trying to figure out the best way to structure one.

A typical – and effective – essay structure might look something like this:

Paragraph #1 (Introduction)

Restate the argument in your own words. Point out the ways in which the argument is flawed – does it paint a misleading picture of the situation? Does it incorrectly use terminology or rely upon leaps of logic to reach its conclusion? Does it rely upon unsubstantiated assumptions?

After identifying the flaws in the argument, state the order in which you plan to examine them.

Paragraphs #2-3 (Body Text)

Restate the first flaw that you felt undermined the argument. Expound upon the flaw and point out why, specifically, this flaw weakens the argument. If possible, offer a way in which this flaw might be ameliorated or avoided.

Paragraph #4 (Improvements) 

If you have the time, suggest a few improvements that would make the argument a stronger one. Explain that in the absence of such improvements, the argument’s flaws are fatal. 

Paragraph #5 (Conclusion)

Conclude by restating the flaws you found and why they weaken the argument irreparably. If appropriate, restate factors that might have strengthened the argument, and explain that in their absence the argument is an unconvincing one.

Get into the habit of structuring your essays in this way, and you’ll soon find that you unconsciously reproduce this format every time you write a practice essay.

Check out our full guide on how to structure your essay to get a 6.0 score!

Practice Sketching Essay Outlines

Once you have your structure in place, you should work on briefly sketching out an outline of your essay before you begin. Using scratch paper (which you’ll have in the actual test), take a minute or two to write out your introduction, body text, improvements and conclusion. Under each one, bullet-point the gist of what you intend to write out.

With the outline written out in brief, you’ll be able to write out your actual essay that much faster.

Take Actual Practice Tests and Stick to the Time Limit

Practice makes perfect, and one of the ways to shake off test-day nerves and nail the AWA is to make it feel like just another day at the office. How can you do that? By replicating test-day conditions and practicing operating within them as often as possible.

There is plenty of practice questions available online that are representative of the kinds of questions you’ll find on the test itself. By practicing these questions, you can give yourself the best possible shot at turning out a 6.0-worthy essay when you finally walk into that test room.

Get Feedback

One of the tricky things about the AWA is that there is no answer key online, so you can’t just do a practice test and get immediate feedback by marking it yourself. Sure, you can try to apply the rubric yourself after the fact, but are you going to be able to do so in an objective and accurate manner? There’s a reason the real thing is marked by both computer and a human examiner (possibly two!). How, then, are you supposed to get feedback?

An excellent way to do this is to submit your practice essays on GMAT forums such as this one. By sharing your answers with both people are have already passed their GMAT and your peers, you’ll be able to get feedback on your essays that you simply wouldn’t be able to give yourself. It can help you to identify weaknesses in your own essay-writing approach and be of use in tightening up your responses.

Develop Your Writing Style

When it comes to writing clearly and accurately, something that can help a lot is avoiding repetition and mixing things up as often as you can. Use synonyms instead of repeating the same words over and over (for instance, instead of repeating ‘strong’, use words like ‘persuasive’, ‘powerful’ or ‘potent’), and try to mix up sentence structures as often as possible.

If you can hit upon a fresh and engaging writing style, you’ll make the assessor more engaged by what you’re saying – and, in turn, ensure that you bump up your scores in that ‘linguistic ability’ portion of the rubric.

Don’t Make Common AWA Errors

Some AWA candidates end up making the same mistakes made by the very arguments they’re critiquing. Faulty or weak assumptions, vague language, improperly made comparisons, causation vs. correlation fallacies, and overly confident conclusions that are not supported by evidence are just a few of the pitfalls into which some candidates can fall.

Make sure you are not one of them by properly supporting any arguments you make and avoiding logical fallacies. It’s also important not to overstate any conclusions you reach; a conclusion is only as strong as the evidence supporting it.

Proofread When You’ve Finished

Try to make sure that you have a few minutes left over at the end to go over your essay and check for spelling, punctuation and grammar errors . It doesn’t matter how good your written English is; we all make mistakes, and we’re more likely to make mistakes when we’re writing quickly and under pressure. A couple of minutes spent reading over your responses can make a huge difference in your overall score, so be sure to incorporate it into your routine!

When preparing for the AWA, it’s very easy to feel overwhelmed and make this section of the GMAT feel like an obstacle that’s simply insurmountable for you. While this is understandable, it’s avoidable if you follow the advice outlined in this article, and prepare yourself for the AWA by developing good habits and getting plenty of practice in.

If you can do this, then you’ll quickly find that the AWA is just one more section to the GMAT – and, like any other section, sufficient preparation removes a lot of the scariness from the equation.

Can You Retake The GRE? Review Of The GRE Retake Policy

How to structure your gmat awa essay for maximum impact, you may also like, gmat awa essay writing: common mistakes and how to avoid them, gmat awa writing tips – how to get 6.0 on awa, the importance of gmat awa essay practice and how to get the most out of..., the gmat awa scoring system: how it works and how to interpret your results, the top 10 tips for acing the gmat awa essay.

  • Skip to content

GMAT Prep Courses & Tutoring

GMAT AWA Topics – Sample Prompts

Before you start.

Remember that the AWA is testing your ability to effectively critique an argument being presented. There will be some common fallacies used in the prompts that you will need to address. Familiarize yourself with these flawed arguments so that you can quickly identify them as you write your essay:

Inappropriate Comparisons – comparing two things which are not necessarily similar, thus leading to a flawed conclusion. You will notice that in an inappropriate comparison, the argument will only note the similarities between two things with no explanation of why the differences do not change the outcome.

Example: "Dogs are highly motivated by food, which is a useful tool for training. Using this same method, you can help to encourage your children to repeat positive behaviors." This argument does not address early childhood development, nor does it address dog psychology – the person assumes that if a solution worked in one situation, it will work in the other.

Vague Language – using words such as "many", "few", "some", "more", or "less" without further qualification is often too vague to make a cogent point.

Example: "In Chicago, more people voted to add bike lanes than people did in Aurora, so it is clear that the citizens in Chicago are more pro-bike than the citizens in Aurora." This argument does not explain what 'more' means. Is it by percentage? Does it refer to raw number of votes, and if so, does it take into account the number of voters in Chicago vs. the number of voters in Aurora?

Correlation vs Causation – this common fallacy asserts that because something occurred, it must be a consequence of some specific variable observed before the result.

Example: "Students in Neighborhood A have higher test scores on average that students in Neighborhood B. Therefore, it is clear that the schools in Neighborhood A are better than the schools in Neighborhood B." This argument is flawed because it assumes the only meaningful variable in test scores was the school the children attend. This argument does not consider other various factors, such as poverty, school resources, home situations, and the parents' abilility to supplement educational opportunities, etc.

Sampling Issues – a case in which statistics are used inappropriately to compare unlike populations or to draw broad conclusions using a small sample.

Example: "56% of the reviews of Company A are negative, which shows that most customers are very unhappy with the service provided." The problem with this argument is that it does not address the percentage of overall customers who have written reviews. How do we know that this is representative of all customers vs. those who are upset about an issue and take the time to write a review?

Now that you have reviewed some common flawed arguments you may encounter, it is time to get started practicing!

Sample Prompt 1

The following appeared in the health section of a magazine on trends and lifestyles:

"People who use the artificial sweetener aspartame are better off consuming sugar, since aspartame can actually contribute to weight gain rather than weight loss. For example, high levels of aspartame have been shown to trigger a craving for food by depleting the brain of a chemical that registers satiety, or the sense of being full. Furthermore, studies suggest that sugars, if consumed after at least 45 minutes of continuous exercise, actually enhance the body’s ability to burn fat. Consequently, those who drink aspartame-sweetened juices after exercise will also lose this calorie-burning benefit. Thus, it appears that people consuming aspartame rather than sugar are unlikely to achieve their dietary goals."

Discuss how well reasoned you find this argument. In your discussion, be sure to analyze the line of reasoning and the use of evidence in the argument. For example, you may need to consider what questionable assumptions underlie the thinking and what alternative explanations or counterexamples might weaken the conclusion. You can also discuss what sort of evidence would strengthen or refute the argument, what changes in the argument would make it more logically sound, and what, if anything, would help you better evaluate its conclusion.

Sample Prompt 2

The following appeared in the opinion column of a financial magazine:

"On average, middle-aged consumers devote 39 percent of their retail expenditure to department store products and services, while for younger consumers the average is only 25 percent. Since the number of middle-aged people will increase dramatically within the next decade, department stores can expect retail sales to increase significantly during that period. Furthermore, to take advantage of the trend, these stores should begin to replace some of those products intended to attract the younger consumer with products intended to attract the middle-aged consumer."

Sample Prompt 3

The following appeared as part of an editorial in an industry newsletter:

"While trucking companies that deliver goods pay only a portion of highway maintenance costs and no property tax on the highways they use, railways spend billions per year maintaining and upgrading their facilities. The government should lower the railroad companies’ property taxes, since sending goods by rail is clearly a more appropriate mode of ground transportation than highway shipping. For one thing, trains consume only a third of the fuel a truck would use to carry the same load, making them a more cost-effective and environmentally sound mode of transport. Furthermore, since rail lines already exist, increases in rail traffic would not require building new lines at the expense of taxpaying citizens."

Sample Prompt 4

The following was excerpted from the speech of a spokesperson for Synthetic Farm Products, Inc.:

"Many farmers who invested in the equipment needed to make the switch from synthetic to organic fertilizers and pesticides feel that it would be too expensive to resume synthetic farming at this point. But studies of farmers who switched to organic farming last year indicate that their current crop yields are lower. Hence their purchase of organic farming equipment, a relatively minor investment compared to the losses that would result from continued lower crop yields, cannot justify persisting on an unwise course. And the choice to farm organically is financially unwise, given that it was motivated by environmental rather than economic concerns."

Sample Prompt 5

The following is part of a business plan created by the management of the Megamart grocery store:

"Our total sales have increased this year by 20 percent since we added a pharmacy section to our grocery store. Clearly, the customer’s main concern is the convenience afforded by one-stop shopping. The surest way to increase our profits over the next couple of years, therefore, is to add a clothing department along with an automotive supplies and repair shop. We should also plan to continue adding new departments and services, such as a restaurant and a garden shop, in subsequent years. Being the only store in the area that offers such a range of services will give us a competitive advantage over other local stores."

GMAT logo

GMAT AWA Practice Tests: Check GMAT Sample Essay Questions PDF

  • GMAT Focus Edition
  • GMAT Question Paper
  • GMAT Quant Practice Paper
  • GMAT Verbal Practice Test
  • GMAT IR Practice Tests
  • GMAT Sentence Correction
  • GMAT Verbal
  • GMAT Quantitative
  • GMAT Geometry
  • GMAT Algebra
  • GMAT Critical Reasoning
  • GMAT Data Sufficiency
  • GMAT ReadingComprehension
  • Integrated Reasoning GMAT
  • GMAT Exam Pattern
  • GMAT Grammar
  • GMAT Coaching Hyderabad
  • GMAT Coaching in Delhi
  • GMAT Coaching In Kolkata
  • GMAT Classes in Mumbai
  • GMAT Preparation Books
  • GMAT Classes in Ahmedabad
  • GMAT Online Coaching
  • GMAT Coaching Bangalore
  • GMAT Study Plan
  • GMAT Score Validity
  • GMAT Score Calculator
  • Cut Off Indian Colleges
  • GMAT Colleges in USA
  • GMAT Cutoff for IIM
  • GMAT Eligibility
  • GMAT Exam Dates
  • GMAT Scholarship
  • GMAT Test Centers

gmat awa essays with answers pdf

Zollege Team

Content Curator | Updated On - Nov 4, 2023

The GMAT AWA practice assesses an aspirant's ability to explain the reasoning behind a given argument and offer a critique in 30 minutes through an essay. The GMAT AWA practice papers include multiple essays that are comparable to those seen on the actual exam. According to GMAT AWA preparation tactics, practicing from example questions is the most beneficial method. Universities frequently use the GMAT AWA or essay scoring to assess candidates' written proficiency. Candidates can see the faults they make while planning, developing, and grammatically correcting their essays by practicing GMAT AWA evaluation PDFs on a daily basis.

GMAT AWA Sample Questions

Here are the GMAT AWA practice test to ace the GMAT preparation .

Universities often use the GMAT AWA or essay scoring to assess candidates' written proficiency. Candidates can see the faults they make while planning, developing, and grammatically correcting their essays by practicing GMAT AWA evaluation PDFs on a daily basis.​

*The article might have information for the previous academic years, please refer the official website of the exam.

Ask your question

UNIVERSITIES/COLLEGES IN USA

UNIVERSITIES/COLLEGES IN CANADA

UNIVERSITIES/COLLEGES IN UK

GMAT Overview

GMAT GMAT Question Paper

GMAT GMAT Syllabus

GMAT GMAT Preparation

GMAT GMAT Score

GMAT GMAT Cut Off

GMAT GMAT Registration

You have found 1 out of 14 Easter Egg decorations! Find them all And get a prize from GMAT Club Club:

5

🔴   Live on YouTube!    Life at UVA Darden - Unfiltered Stories by 3 GMAT Club Members at Darden

Close

Customized for You

Track Your Progress

Practice Pays

It appears that you are browsing the GMAT Club forum unregistered!

Take 11 tests and quizzes from GMAT Club and leading GMAT prep companies such as Manhattan Prep. All are free for GMAT Club members.

View detailed applicant stats such as GPA, GMAT score, work experience, location, application status, and more

Download thousands of study notes, question collections, GMAT Club’s Grammar and Math books. All are free!

  • Register now! It`s easy!
  • Already registered? Sign in!

Thank you for using the timer! We noticed you are actually not timing your practice. Click the START button first next time you use the timer. There are many benefits to timing your practice , including:

We’ll give you an estimate of your score

We’ll provide personalized question recommendations

Your score will improve and your results will be more realistic

gmat awa essays with answers pdf

GMAT RC Masterclass

Get an Extra 20% Off Target Test Prep GMAT Focus Plans

3rd Annual Easter Egg Hunt - Continues

Master Hard MSR Questions on GMAT Focus (Free Webinar)

Inside the UVA Darden MBA Experience: Unfiltered Stories from Current Students | Bootcamp Mentality

GMAT Online Course at $159 | 2X more Comprehensive Content for GMAT Focus Edition

How to Get US Student Visa | F1 Visa Process, Interview Questions, Proof of Funding, I-20, SEVIS

­The GMAT Club Podcast - What happens inside the Business School Decision Room?

GMAT vs. EA: Which is right for you?

gmat awa essays with answers pdf

07:30 PM IST

09:30 PM IST

gmat awa essays with answers pdf

10:00 AM EDT

11:59 PM EDT

gmat awa essays with answers pdf

12:00 PM PDT

11:59 PM PDT

gmat awa essays with answers pdf

05:30 AM PDT

07:30 AM PDT

07:00 AM PDT

08:00 AM PDT

01:30 PM IST

11:30 PM IST

08:30 AM PDT

09:30 AM PDT

10:00 AM PDT

11:00 AM PDT

08:00 PM EDT

09:00 PM EDT

Sample AWA Prompt and 6/6 Essay

User avatar

  • GMAT Prep Courses
  • MBA Programs
  • Build Your Study Plan
  • Best GMAT Books
  • All the GMAT Tests
  • GMAT Focus Tests
  • GMAT Verbal
  • Error Log Templates
  • GMAT Official Guide
  • Manhattan Prep
  • Target Test Prep
  • Experts' Global
  • Full Time MBA Rankings
  • Part Time MBA Rankings
  • Executive MBA Rankings
  • International MBA Rankings
  • Best App Tips
  • Best MBA Books
  • Application Reference
  • Free Profile Evaluation

Copyright © 2024 GMAT Club

GMAT ® is a registered trademark of the Graduate Management Admission Council ® (GMAC ®). GMAT Club's website has not been reviewed or endorsed by GMAC.

gmat awa essays with answers pdf

The post is bookmarked successfully

IMAGES

  1. Gmat Awa Sample Essays With Answers Pdf

    gmat awa essays with answers pdf

  2. GMAT AWA

    gmat awa essays with answers pdf

  3. GMAT Writing Section Examples

    gmat awa essays with answers pdf

  4. Gmat Awa Sample Essays With Answers Pdf

    gmat awa essays with answers pdf

  5. GMAT Analytical Writing: All About the GMAT Essay and How to Prepare

    gmat awa essays with answers pdf

  6. Gmat AWA Template

    gmat awa essays with answers pdf

VIDEO

  1. GMAT Tuesday: AWA

  2. 4

  3. Live Walkthrough

  4. GMAT Complete Course

  5. 5

  6. 2

COMMENTS

  1. REVIEWER: AWA Solutions (100+ essays, with answers)

    Mar 25. How Conner Scored GMAT Focus 755 (100%) 12:00 PM EDT. -. 01:00 PM EDT. After just 3 months of studying with the TTP GMAT Focus course, Conner scored an incredible 755 (Q89/V90/DI83) on the GMAT Focus. In this live interview, he shares how he achieved his outstanding 755 (100%) GMAT Focus score on test day.

  2. REVIEWER: AWA Solutions (100+ essays, with answers) : General GMAT

    AWA Solutions (Sample Essays).pdf [1.36 MiB] Downloaded 29099 times. ... AWA Solutions (100+ essays, use answers) ... Is article shares 50 magazine on GMAT essay with GMAT AWA. We also have other articles for GMAT essay tips up help they score perfect 6 on the AWA fachgruppe.

  3. Example of a High-Scoring AWA Essay

    Sample AWA Prompt (Fern Valley University) The following appeared as part of a recommendation from the financial planning office to the administration of Fern Valley University. "In the past few years, Fern Valley University has suffered from a decline in both enrollments and admissions applications. The reason can be discovered from our ...

  4. GMAT AWA Example Essay

    This is a particular long and thorough sample essay, but it gives you an idea of what it takes to get a 6. In line with the AWA directions, notice that I organized, developed, and expressed my ideas about the argument presented. I provided relevant supporting reasons and examples --- i.e. I didn't just say, "This is bad," but I provided a ...

  5. GMAT Essay: Format, Scoring, and Tips for the AWA

    The GMAT Analytical Writing Assessment requires you to write an essay, using a basic word processor, that critiques a provided argument. It is the first section of the GMAT and is to be completed in 30 minutes. The AWA is scored by two readers in half-intervals on a scale of 0-6.

  6. GMAT Essay

    GMAT Essay - List of AWA Topics - 50 Practice Questions on GMAT AWA 2023. This document contains practice questions that will help you improve the AWA section or the GMAT essay section. Discuss in your GMAT essay how well-reasoned you find the argument. Here are some ways you can do that while writing your GMAT essay for the AWA section.

  7. PDF Analysis of an Argument Questions for the GMAT® Exam

    Microsoft Word - mba.com file - Analysis of an Argument 100606.doc. This document contains most Analysis of an Argument questions used on the GMAT® exam. Each question is followed by this statement: Discuss how well reasoned you find this argument. In your discussion be sure to analyze the line of reasoning and the use of evidence in the argument.

  8. GMAT Analytical Writing: All About the GMAT Essay and How ...

    Once you've read few a through sample AWA prompts, read through the third prompt on page 31 of the PDF. Magoosh GMAT expert Mike McGarry has written a great GMAT AWA Example essay in response to this prompt, including analysis of why it works well and why it would receive a 6. Scoring for GMAT Analytical Writing

  9. The Best GMAT Essay Template to Help You Ace the AWA

    An effective GMAT essay template will include the following features: The structure of your essay, paragraph by paragraph. The kind of content that should be in each paragraph. Varied and sophisticated pre-written sentence stems (as in, the beginning part of a sentence) for the main kinds of flaws.

  10. GMAT Analytical Writing Assessment (AWA) Section

    Introduction to the GMAT AWA Task. The Analytical Writing section on the GMAT (often referred to simply as the AWA) is possibly the one with most diametric opinions. Some people actually look forward to the opportunity to break out of the multiple-choice format and to present their thoughts on paper, while others are terrified by the prospect.

  11. The 6 GMAT Essay Tips That Will Help You Ace the AWA

    Another one of the important GMAT writing tips is to take the time to set up your essay in a clear way. You don't need to write the most interesting or lengthy essay in the world to score well on the AWA section, but you do need to give your essay an easy-to-follow structure. Usually, that consists of an introduction, three to four well ...

  12. GMAT AWA

    As I mentioned, for the Analytical Writing Assessment (AWA) on the GMAT, students are presented with an argument and are asked to evaluate it. Students need to break down the argument, point out weaknesses and gaps in the reasoning and examples, and suggest ways to improve the argument. All this needs to be accomplished in thirty minutes.

  13. GMAT Analytical Writing Assessment tips

    Here are a few tips to prepare you for the GMAT AWA section. Step 1 - Understand the process of creating a well-rounded analysis of an argument. You can do that in two ways or rather with the help of two tools. Step 2 - Practice! Practice! Practice! the AWA questions.

  14. GMAT Analytical Writing: All About the GMAT Essay and How to ...

    Use a GMAT Writing Template: Many test-takers find it helpful to follow a basic GMAT AWA template structure and practice it in advance, so it's just a matter of plugging in the specific details on test day. Here's an example of a possible GMAT writing template. Feel free to adapt this writing template as-is, modify it, or create one of your ...

  15. How to write your GMAT AWA Essay [Effectively]

    2. On the AWA, you need to be a lawyer, that is, you need to find faults with the given argument. You do not need to be a journalist, that is, you do not need to write about all possible perspectives of an issue. Also remember, that this is an analysis, not an opinion piece. Do not bring YOUR perspective and your opinions into the essay.

  16. AWA compilations

    The AWA lends itself very easily to the use of an essay template. No matter what the argument prompt is, you can bet that there will be at least 3 glaring errors of logic in it. You can, therefore, use a template to structure your AWA essay. Using a template takes most of the stress away from the AWA section.

  17. GMAT AWA Sample Questions 2022 and Practice Questions

    GMAT analytical writing assessment tests the aspirant's capability to rationalize the reasoning behind the given argument and produce a critique through an essay in 30 minutes. The GMAT AWA practice papers contain numerous essays that are similar to the actual exam. GMAT AWA preparation strategies consider practicing from sample questions the ...

  18. The GMAT AWA: Understanding the Format and How to Prepare

    The GMAT AWA: Understanding the Format and How to Prepare. March 29, 2023 325 views 14 minutes read. The GMAT AWA consists of a single long-form essay, and is an assessment of your ability to analyze and critique a flawed argument. It can be quite an intimidating section for many GMAT candidates, particularly those who are non-native speakers.

  19. PDF 12 EXCLUSIVE GMAT EXAM QUESTIONS

    GMAT® Exam At-A-Glance 1 Essay 12 Questions 31 Questions 36 Questions ... The Analytical Writing Assessment (AWA) measures your ability to think critically and to communicate your ideas. For this exam question, you are asked to analyze the reasoning ... Answer Choices A. 901 B. 989 C. 990 D. 991 E. 1,001 3 4 Quantitative Reasoning PROBLEM SOLVING

  20. All AWA Topics for GMAT Collection (Official List from GMAC)

    All Official AWA Topics. Attached are the latest/current collection of all of the Analysis of an Argument Essay/AWA (Analytical Writing Assessment) Topics administered during the GMAT. Unlike GMAT Questions, GMAC actually publishes the full list of all possible Essay Questions - in other words - these are the only topics you will be presented with.

  21. GMAT AWA Topics

    Sample Prompt 5. The following is part of a business plan created by the management of the Megamart grocery store: "Our total sales have increased this year by 20 percent since we added a pharmacy section to our grocery store. Clearly, the customer's main concern is the convenience afforded by one-stop shopping.

  22. GMAT AWA Practice Tests: Check GMAT Sample Essay Questions PDF

    The GMAT AWA practice papers include multiple essays that are comparable to those seen on the actual exam. According to GMAT AWA preparation tactics, practicing from example questions is the most beneficial method. Universities frequently use the GMAT AWA or essay scoring to assess candidates' written proficiency. Candidates can see the faults ...

  23. Sample AWA Prompt and 6/6 Essay

    The essay demonstrates good coherence and connectivity. Ideas are logically organized, and there is a clear flow between sentences and paragraphs. The transitions help guide the reader through the analysis effectively. Word Structure: 5/6 The word structure is generally clear, and the essay uses a variety of sentence structures.